Daily Drills 67 - Section 67 - Question 1
The conclusion of the columnist's argument can be properly drawn if which one of the following is assumed?
Replies
Irina November 10, 2019
@nimakian,This question stem indicates a strengthen with sufficient assumption question. A sufficient assumption allows us to draw a conclusion/ guarantees the truth of the conclusion, whereas in a must be true question we are required to make an inference/ reach a conclusion assuming all the premises are true.
Other question stems that indicate a sufficient assumption question include:
"The conclusion above follows logically if which one of the following is assumed?â€
“Which of the following, if true, enables the conclusion to be properly drawn?â€
“The conclusion above is properly drawn if which of the following is assumed?â€
nimakian801 November 10, 2019
Oh I see. So it's a different between the statements in the passage making something true versus something in the answer choice if true helps us conclude the above. Did I get this right?
Irina November 11, 2019
@nimakian801,You got it!
Starr-Latimer February 5, 2023
Why wouldn't this be a strengthen with necessary question? I thought the "must be assumed" implied a necessary factor.
Emil-Kunkin February 5, 2023
I don't see any language about must be assumed, which would indeed make this a necessary assumption. This is asking us what would make it so the argument can be valid, which is a sufficient assumption.